Troubleshooting Homework: Identifying and Addressing Mistakes

  • Thread starter Thread starter Shackleford
  • Start date Start date
Click For Summary
The discussion focuses on troubleshooting a homework problem related to a conservative field and path independence. Participants emphasize the importance of recognizing that the integral of a gradient function is key to finding the solution. There is confusion regarding the final answer, with one participant calculating a result of 3/4, while the textbook answer is -27/4. The conversation suggests that the original poster has not fully completed the problem, which is crucial for arriving at the correct answer. Overall, the key takeaway is the necessity of finishing the calculations to resolve discrepancies in the answers.
Shackleford
Messages
1,649
Reaction score
2

Homework Statement



See Image

Homework Equations



See Image

The Attempt at a Solution



See Image
 

Attachments

  • review.jpg
    review.jpg
    20.1 KB · Views: 488
Physics news on Phys.org
You're right, it is a conservative field - that's the key to the solution.
I think you're trying to use path independence, which is also key, but try it another way.
you're looking for the integral of F; and F = grad f... well... what's the integral of grad f?
 
Actually, I can't really see a problem. So...am I really just that rusty? What's the answer supposed to be?
 
The only thing you are doing wrong is that you have not finished the problem!

You have (9- 12+ 4)- (1/4+ 1- 1). Okay, what number is that?
 
HallsofIvy said:
The only thing you are doing wrong is that you have not finished the problem!

You have (9- 12+ 4)- (1/4+ 1- 1). Okay, what number is that?

Well, that answer is 3/4, but the answer in the back of the review has -27/4.
 
lzkelley said:
You're right, it is a conservative field - that's the key to the solution.
I think you're trying to use path independence, which is also key, but try it another way.
you're looking for the integral of F; and F = grad f... well... what's the integral of grad f?

I have that written out.
 
Question: A clock's minute hand has length 4 and its hour hand has length 3. What is the distance between the tips at the moment when it is increasing most rapidly?(Putnam Exam Question) Answer: Making assumption that both the hands moves at constant angular velocities, the answer is ## \sqrt{7} .## But don't you think this assumption is somewhat doubtful and wrong?

Similar threads

  • · Replies 17 ·
Replies
17
Views
3K
  • · Replies 5 ·
Replies
5
Views
2K
  • · Replies 2 ·
Replies
2
Views
2K
  • · Replies 7 ·
Replies
7
Views
2K
Replies
5
Views
2K
  • · Replies 28 ·
Replies
28
Views
3K
Replies
8
Views
2K
  • · Replies 7 ·
Replies
7
Views
2K
  • · Replies 16 ·
Replies
16
Views
2K
  • · Replies 7 ·
Replies
7
Views
2K